The Stacks project

57.13 Fully faithful functors

Our goal is to prove fully faithful functors between derived categories are siblings of Fourier-Mukai functors, following [Orlov-K3] and [Ballard].

Situation 57.13.1. Here $k$ is a field. We have proper smooth schemes $X$ and $Y$ over $k$. We have a $k$-linear, exact, fully faithful functor $F : D_{perf}(\mathcal{O}_ X) \to D_{perf}(\mathcal{O}_ Y)$.

Before reading on, it makes sense to read at least some of Derived Categories, Section 13.41.

Recall that $X$ is regular and hence has the resolution property (Varieties, Lemma 33.25.3 and Derived Categories of Schemes, Lemma 36.36.8). Thus on $X \times X$ we may choose a resolution

\[ \ldots \to \mathcal{E}_2 \boxtimes \mathcal{G}_2 \to \mathcal{E}_1 \boxtimes \mathcal{G}_1 \to \mathcal{E}_0 \boxtimes \mathcal{G}_0 \to \mathcal{O}_\Delta \to 0 \]

where each $\mathcal{E}_ i$ and $\mathcal{G}_ i$ is a finite locally free $\mathcal{O}_ X$-module, see Lemma 57.9.3. Using the complex

57.13.1.1
\begin{equation} \label{equiv-equation-original-complex} \ldots \to \mathcal{E}_2 \boxtimes \mathcal{G}_2 \to \mathcal{E}_1 \boxtimes \mathcal{G}_1 \to \mathcal{E}_0 \boxtimes \mathcal{G}_0 \end{equation}

in $D_{perf}(\mathcal{O}_{X \times X})$ as in Derived Categories, Example 13.41.2 if for each $n$ we denote

\[ M_ n = (\mathcal{E}_ n \boxtimes \mathcal{G}_ n \to \ldots \to \mathcal{E}_0 \boxtimes \mathcal{G}_0)[-n] \]

we obtain an infinite Postnikov system for the complex (57.13.1.1). This means the morphisms $M_0 \to M_1[1] \to M_2[2] \to \ldots $ and $M_ n \to \mathcal{E}_ n \boxtimes \mathcal{G}_ n$ and $\mathcal{E}_ n \boxtimes \mathcal{G}_ n \to M_{n - 1}$ satisfy certain conditions documented in Derived Categories, Definition 13.41.1. Set

\[ \mathcal{F}_ n = \mathop{\mathrm{Ker}}(\mathcal{E}_ n \boxtimes \mathcal{G}_ n \to \mathcal{E}_{n - 1} \boxtimes \mathcal{G}_{n - 1}) \]

Observe that since $\mathcal{O}_\Delta $ is flat over $X$ via $\text{pr}_1$ the same is true for $\mathcal{F}_ n$ for all $n$ (this is a convenient though not essential observation). We have

\[ H^ q(M_ n[n]) = \left\{ \begin{matrix} \mathcal{O}_\Delta & \text{if} & q = 0 \\ \mathcal{F}_ n & \text{if} & q = -n \\ 0 & \text{if} & q \not= 0, -n \end{matrix} \right. \]

Thus for $n \geq \dim (X \times X)$ we have

\[ M_ n[n] \cong \mathcal{O}_\Delta \oplus \mathcal{F}_ n[n] \]

in $D_{perf}(\mathcal{O}_{X \times X})$ by Lemma 57.9.5.

We are interested in the complex

57.13.1.2
\begin{equation} \label{equiv-equation-complex} \ldots \to \mathcal{E}_2 \boxtimes F(\mathcal{G}_2) \to \mathcal{E}_1 \boxtimes F(\mathcal{G}_1) \to \mathcal{E}_0 \boxtimes F(\mathcal{G}_0) \end{equation}

in $D_{perf}(\mathcal{O}_{X \times Y})$ as the “totalization” of this complex should give us the kernel of the Fourier-Mukai functor we are trying to construct. For all $i, j \geq 0$ we have

\begin{align*} \mathop{\mathrm{Ext}}\nolimits ^ q_{X \times Y}(\mathcal{E}_ i \boxtimes F(\mathcal{G}_ i), \mathcal{E}_ j \boxtimes F(\mathcal{G}_ j)) & = \bigoplus \nolimits _ p \mathop{\mathrm{Ext}}\nolimits ^{q + p}_ X(\mathcal{E}_ i, \mathcal{E}_ j) \otimes _ k \mathop{\mathrm{Ext}}\nolimits ^{-p}_ Y(F(\mathcal{G}_ i), F(\mathcal{G}_ j)) \\ & = \bigoplus \nolimits _ p \mathop{\mathrm{Ext}}\nolimits ^{q + p}_ X(\mathcal{E}_ i, \mathcal{E}_ j) \otimes _ k \mathop{\mathrm{Ext}}\nolimits ^{-p}_ X(\mathcal{G}_ i, \mathcal{G}_ j) \end{align*}

The second equality holds because $F$ is fully faithful and the first by Derived Categories of Schemes, Lemma 36.25.1. We find these $\mathop{\mathrm{Ext}}\nolimits ^ q$ are zero for $q < 0$. Hence by Derived Categories, Lemma 13.41.6 we can build an infinite Postnikov system $K_0, K_1, K_2, \ldots $ in $D_{perf}(\mathcal{O}_{X \times Y})$ for the complex (57.13.1.2). Parallel to what happens with $M_0, M_1, M_2, \ldots $ this means we obtain morphisms $K_0 \to K_1[1] \to K_2[2] \to \ldots $ and $K_ n \to \mathcal{E}_ n \boxtimes F(\mathcal{G}_ n)$ and $\mathcal{E}_ n \boxtimes F(\mathcal{G}_ n) \to K_{n - 1}$ in $D_{perf}(\mathcal{O}_{X \times Y})$ satisfying certain conditions documented in Derived Categories, Definition 13.41.1.

Let $\mathcal{F}$ be a coherent $\mathcal{O}_ X$-module whose support has a finite number of points, i.e., with $\dim (\text{Supp}(\mathcal{F})) = 0$. Consider the exact functor of triangulated categories

\[ D_{perf}(\mathcal{O}_{X \times Y}) \longrightarrow D_{perf}(\mathcal{O}_ Y),\quad N \longmapsto R\text{pr}_{2, *}(\text{pr}_1^*\mathcal{F} \otimes ^\mathbf {L}_{\mathcal{O}_{X \times Y}} N) \]

It follows that the objects $R\text{pr}_{2, *}(\text{pr}_1^*\mathcal{F} \otimes ^\mathbf {L}_{\mathcal{O}_{X \times Y}} K_ i)$ form a Postnikov system for the complex in $D_{perf}(\mathcal{O}_ Y)$ with terms

\[ R\text{pr}_{2, *}( (\mathcal{F} \otimes \mathcal{E}_ i) \boxtimes F(\mathcal{G}_ i)) = \Gamma (X, \mathcal{F} \otimes \mathcal{E}_ i) \otimes _ k F(\mathcal{G}_ i) = F(\Gamma (X, \mathcal{F} \otimes \mathcal{E}_ i) \otimes _ k \mathcal{G}_ i) \]

Here we have used that $\mathcal{F} \otimes \mathcal{E}_ i$ has vanishing higher cohomology as its support has dimension $0$. On the other hand, applying the exact functor

\[ D_{perf}(\mathcal{O}_{X \times X}) \longrightarrow D_{perf}(\mathcal{O}_ Y),\quad N \longmapsto F(R\text{pr}_{2, *}(\text{pr}_1^*\mathcal{F} \otimes ^\mathbf {L}_{\mathcal{O}_{X \times X}} N)) \]

we find that the objects $F(R\text{pr}_{2, *}(\text{pr}_1^*\mathcal{F} \otimes ^\mathbf {L}_{\mathcal{O}_{X \times X}} M_ n))$ form a second infinite Postnikov system for the complex in $D_{perf}(\mathcal{O}_ Y)$ with terms

\[ F(R\text{pr}_{2, *}( (\mathcal{F} \otimes \mathcal{E}_ i) \boxtimes \mathcal{G}_ i)) = F(\Gamma (X, \mathcal{F} \otimes \mathcal{E}_ i) \otimes _ k \mathcal{G}_ i) \]

This is the same as before! By uniqueness of Postnikov systems (Derived Categories, Lemma 13.41.6) which applies because

\[ \mathop{\mathrm{Ext}}\nolimits ^ q_ Y( F(\Gamma (X, \mathcal{F} \otimes \mathcal{E}_ i) \otimes _ k \mathcal{G}_ i), F(\Gamma (X, \mathcal{F} \otimes \mathcal{E}_ j) \otimes _ k \mathcal{G}_ j)) = 0, \quad q < 0 \]

as $F$ is fully faithful, we find a system of isomorphisms

\[ F(R\text{pr}_{2, *}(\text{pr}_1^*\mathcal{F} \otimes ^\mathbf {L}_{\mathcal{O}_{X \times X}} M_ n[n])) \cong R\text{pr}_{2, *}(\text{pr}_1^*\mathcal{F} \otimes ^\mathbf {L}_{\mathcal{O}_{X \times Y}} K_ n[n]) \]

in $D_{perf}(\mathcal{O}_ Y)$ compatible with the morphisms in $D_{perf}(\mathcal{O}_ Y)$ induced by the morphisms

\[ M_{n - 1}[n - 1] \to M_ n[n] \quad \text{and}\quad K_{n - 1}[n - 1] \to K_ n[n] \]
\[ M_ n \to \mathcal{E}_ n \boxtimes \mathcal{G}_ n \quad \text{and}\quad K_ n \to \mathcal{E}_ n \boxtimes F(\mathcal{G}_ n) \]
\[ \mathcal{E}_ n \boxtimes \mathcal{G}_ n \to M_{n - 1} \quad \text{and}\quad \mathcal{E}_ n \boxtimes F(\mathcal{G}_ n) \to K_{n - 1} \]

which are part of the structure of Postnikov systems. For $n$ sufficiently large we obtain a direct sum decomposition

\[ F(R\text{pr}_{2, *}(\text{pr}_1^*\mathcal{F} \otimes ^\mathbf {L}_{\mathcal{O}_{X \times X}} M_ n[n])) = F(\mathcal{F}) \oplus F(R\text{pr}_{2, *}( \text{pr}_1^*\mathcal{F} \otimes _{\mathcal{O}_{X \times Y}} \mathcal{F}_ n ))[n] \]

corresponding to the direct sum decomposition of $M_ n$ constructed above (we are using the flatness of $\mathcal{F}_ n$ over $X$ via $\text{pr}_1$ to write a usual tensor product in the formula above, but this isn't essential for the argument). By Lemma 57.9.9 we find there exists an integer $m \geq 0$ such that the first summand in this direct sum decomposition has nonzero cohomology sheaves only in the interval $[-m, m]$ and the second summand in this direct sum decomposition has nonzero cohomology sheaves only in the interval $[-m - n, m + \dim (X) - n]$. We conclude the system $K_0 \to K_1[1] \to K_2[2] \to \ldots $ in $D_{perf}(\mathcal{O}_{X \times Y})$ satisfies the assumptions of Lemma 57.9.10 after possibly replacing $m$ by a larger integer. We conclude we can write

\[ K_ n[n] = K \oplus C_ n \]

for $n \gg 0$ compatible with transition maps and with $C_ n$ having nonzero cohomology sheaves only in the range $[-m - n, m - n]$. Denote $G$ the Fourier-Mukai functor corresponding to $K$. Putting everything together we find

\[ \begin{matrix} G(\mathcal{F}) \oplus R\text{pr}_{2, *}( \text{pr}_1^*\mathcal{F} \otimes _{\mathcal{O}_{X \times Y}}^\mathbf {L} C_ n) \cong \\ R\text{pr}_{2, *}(\text{pr}_1^*\mathcal{F} \otimes ^\mathbf {L}_{\mathcal{O}_{X \times Y}} K_ n[n]) \cong \\ F(R\text{pr}_{2, *}(\text{pr}_1^*\mathcal{F} \otimes ^\mathbf {L}_{\mathcal{O}_{X \times X}} M_ n[n])) \cong \\ F(\mathcal{F}) \oplus F(R\text{pr}_{2, *}( \text{pr}_1^*\mathcal{F} \otimes _{\mathcal{O}_{X \times Y}} \mathcal{F}_ n ))[n] \end{matrix} \]

Looking at the degrees that objects live in we conclude that for $n \gg m$ we obtain an isomorphism

\[ F(\mathcal{F}) \cong G(\mathcal{F}) \]

Moreover, recall that this holds for every coherent $\mathcal{F}$ on $X$ whose support has dimension $0$.

Lemma 57.13.2. Let $k$ be a field. Let $X$ and $Y$ be smooth proper schemes over $k$. Given a $k$-linear, exact, fully faithful functor $F : D_{perf}(\mathcal{O}_ X) \to D_{perf}(\mathcal{O}_ Y)$ there exists a Fourier-Mukai functor $F' : D_{perf}(\mathcal{O}_ X) \to D_{perf}(\mathcal{O}_ Y)$ whose kernel is in $D_{perf}(\mathcal{O}_{X \times Y})$ which is a sibling to $F$.

Proof. Apply Lemma 57.12.6 to $F$ and the functor $G$ constructed above. $\square$

The following theorem is also true without assuming $X$ is projective, see [Noah].

reference

Theorem 57.13.3 (Orlov). Let $k$ be a field. Let $X$ and $Y$ be smooth proper schemes over $k$ with $X$ projective over $k$. Any $k$-linear fully faithful exact functor $F : D_{perf}(\mathcal{O}_ X) \to D_{perf}(\mathcal{O}_ Y)$ is a Fourier-Mukai functor for some kernel in $D_{perf}(\mathcal{O}_{X \times Y})$.

Proof. Let $F'$ be the Fourier-Mukai functor which is a sibling of $F$ as in Lemma 57.13.2. By Proposition 57.10.6 we have $F \cong F'$ provided we can show that $\textit{Coh}(\mathcal{O}_ X)$ has enough negative objects. However, if $X = \mathop{\mathrm{Spec}}(k)$ for example, then this isn't true. Thus we first decompose $X = \coprod X_ i$ into its connected (and irreducible) components and we argue that it suffices to prove the result for each of the (fully faithful) composition functors

\[ F_ i : D_{perf}(\mathcal{O}_{X_ i}) \to D_{perf}(\mathcal{O}_ X) \to D_{perf}(\mathcal{O}_ Y) \]

Details omitted. Thus we may assume $X$ is irreducible.

The case $\dim (X) = 0$. Here $X$ is the spectrum of a finite (separable) extension $k'/k$ and hence $D_{perf}(\mathcal{O}_ X)$ is equivalent to the category of graded $k'$-vector spaces such that $\mathcal{O}_ X$ corresponds to the trivial $1$-dimensional vector space in degree $0$. It is straightforward to see that any two siblings $F, F' : D_{perf}(\mathcal{O}_ X) \to D_{perf}(\mathcal{O}_ Y)$ are isomorphic. Namely, we are given an isomorphism $F(\mathcal{O}_ X) \cong F'(\mathcal{O}_ X)$ compatible the action of the $k$-algebra $k' = \text{End}_{D_{perf}(\mathcal{O}_ X)}(\mathcal{O}_ X)$ which extends canonically to an isomorphism on any graded $k'$-vector space.

The case $\dim (X) > 0$. Here $X$ is a projective smooth variety of dimension $> 1$. Let $\mathcal{F}$ be a coherent $\mathcal{O}_ X$-module. We have to show there exists a coherent module $\mathcal{N}$ such that

  1. there is a surjection $\mathcal{N} \to \mathcal{F}$ and

  2. $\mathop{\mathrm{Hom}}\nolimits (\mathcal{F}, \mathcal{N}) = 0$.

Choose an ample invertible $\mathcal{O}_ X$-module $\mathcal{L}$. We claim that $\mathcal{N} = (\mathcal{L}^{\otimes n})^{\oplus r}$ will work for $n \ll 0$ and $r$ large enough. Condition (1) follows from Properties, Proposition 28.26.13. Finally, we have

\[ \mathop{\mathrm{Hom}}\nolimits (\mathcal{F}, \mathcal{L}^{\otimes n}) = H^0(X, \mathop{\mathcal{H}\! \mathit{om}}\nolimits (\mathcal{F}, \mathcal{L}^{\otimes n})) = H^0(X, \mathop{\mathcal{H}\! \mathit{om}}\nolimits (\mathcal{F}, \mathcal{O}_ X) \otimes \mathcal{L}^{\otimes n}) \]

Since the dual $\mathop{\mathcal{H}\! \mathit{om}}\nolimits (\mathcal{F}, \mathcal{O}_ X)$ is torsion free, this vanishes for $n \ll 0$ by Varieties, Lemma 33.48.1. This finishes the proof. $\square$

Proposition 57.13.4. Let $k$ be a field. Let $X$ and $Y$ be smooth proper schemes over $k$. If $F : D_{perf}(\mathcal{O}_ X) \to D_{perf}(\mathcal{O}_ Y)$ is a $k$-linear exact equivalence of triangulated categories then there exists a Fourier-Mukai functor $F' : D_{perf}(\mathcal{O}_ X) \to D_{perf}(\mathcal{O}_ Y)$ whose kernel is in $D_{perf}(\mathcal{O}_{X \times Y})$ which is an equivalence and a sibling of $F$.

Proof. The functor $F'$ of Lemma 57.13.2 is an equivalence by Lemma 57.10.3. $\square$

Lemma 57.13.5. Let $k$ be a field. Let $X$ be a smooth proper scheme over $k$. Let $K \in D_{perf}(\mathcal{O}_{X \times X})$. If the Fourier-Mukai functor $\Phi _ K : D_{perf}(\mathcal{O}_ X) \to D_{perf}(\mathcal{O}_ X)$ is isomorphic to the identity functor, then $K \cong \Delta _*\mathcal{O}_ X$ in $_{perf}(\mathcal{O}_{X \times X})$.

Proof. Let $i$ be the minimal integer such that the cohomology sheaf $H^ i(K)$ is nonzero. Let $\mathcal{E}$ and $\mathcal{G}$ be finite locally free $\mathcal{O}_ X$-modules. Then

\begin{align*} H^ i(X \times X, K \otimes _{\mathcal{O}_{X \times X}}^\mathbf {L} (\mathcal{E} \boxtimes \mathcal{G})) & = H^ i(X, R\text{pr}_{2, *}(K \otimes _{\mathcal{O}_{X \times X}}^\mathbf {L} (\mathcal{E} \boxtimes \mathcal{G}))) \\ & = H^ i(X, \Phi _ K(\mathcal{E}) \otimes _{\mathcal{O}_ X}^\mathbf {L} \mathcal{G}) \\ & \cong H^ i(X, \mathcal{E} \otimes \mathcal{G}) \end{align*}

which is zero if $i < 0$. On the other hand, we can choose $\mathcal{E}$ and $\mathcal{G}$ such that there is a surjection $\mathcal{E}^\vee \boxtimes \mathcal{G}^\vee \to H^ i(K)$ by Lemma 57.9.1. In this case the left hand side of the equalities is nonzero. Hence we conclude that $H^ i(K) = 0$ for $i < 0$.

Let $i$ be the maximal integer such that $H^ i(K)$ is nonzero. The same argument with $\mathcal{E}$ and $\mathcal{G}$ support of dimension $0$ shows that $i \leq 0$. Hence we conclude that $K$ is given by a single coherent $\mathcal{O}_{X \times X}$-module $\mathcal{K}$ sitting in degree $0$.

Since $R\text{pr}_{2, *}(\text{pr}_1^*\mathcal{F} \otimes \mathcal{K})$ is $\mathcal{F}$, by taking $\mathcal{F}$ supported at closed points we see that the support of $\mathcal{K}$ is finite over $X$ via $\text{pr}_2$. Since $R\text{pr}_{2, *}(\mathcal{K}) \cong \mathcal{O}_ X$ we conclude by Functors and Morphisms, Lemma 56.7.6 that $\mathcal{K} = s_*\mathcal{O}_ X$ for some section $s : X \to X \times X$ of the second projection. Then $\Phi _ K(M) = f^*M$ where $f = \text{pr}_1 \circ s$ and this can happen only if $s$ is the diagonal morphism as desired. $\square$


Comments (0)


Post a comment

Your email address will not be published. Required fields are marked.

In your comment you can use Markdown and LaTeX style mathematics (enclose it like $\pi$). A preview option is available if you wish to see how it works out (just click on the eye in the toolbar).

Unfortunately JavaScript is disabled in your browser, so the comment preview function will not work.

All contributions are licensed under the GNU Free Documentation License.




In order to prevent bots from posting comments, we would like you to prove that you are human. You can do this by filling in the name of the current tag in the following input field. As a reminder, this is tag 0G07. Beware of the difference between the letter 'O' and the digit '0'.